0 Daumen
200 Aufrufe

Zeige, dass die Reihe konvergiert und bestimmen Sie ihren Wert.

\( \sum\limits_{k=1}^{\infty}{\frac{1}{k^2}-\frac{1}{(k+1)^2}} \) = \( \sum\limits_{k=1}^{\infty}{\frac{2k+1}{k^2(k+1)^2}} \)


Problem/Ansatz:

Ich glaube zu zeigen das es konvergiert ist noch einfach aber zu zeigen welchen Wert die Reihe annimmt nicht so (also so geht es mir zu mindestens). Ich wollte mittels Teleskopsumme beides zeigen aber scheitere immer an einer Folgendarstellung. Könnte mir jemand helfen?

Avatar von

2 Antworten

+1 Daumen
 
Beste Antwort

Aloha :)

Hier brauchst du eigentlich nicht viel zu rechnen. Betrachte:$$S_n=\sum\limits_{k=1}^n\left(\frac{1}{k^2}-\frac{1}{(k+1)^2}\right)=\sum\limits_{k=1}^n\frac{1}{k^2}-\sum\limits_{k=1}^n\frac{1}{(k+1)^2}$$$$\phantom{S_n}=\sum\limits_{k=1}^n\frac{1}{k^2}-\sum\limits_{k=1\pink{+1}}^{n\pink{+1}}\frac{1}{((k\pink{-1})+1)^2}=\sum\limits_{k=1}^n\frac{1}{k^2}-\sum\limits_{k=2}^{n+1}\frac{1}{k^2}$$$$\phantom{S_n}=\left(\frac{1}{1^2}+\sum\limits_{k=2}^n\frac{1}{k^2}\right)-\left(\sum\limits_{k=2}^n\frac{1}{k^2}+\frac{1}{(n+1)^2}\right)=1-\frac{1}{(n+1)^2}$$

Im Grenzwert \(n\to\infty\) erhältst du als Summe der unendlichen Reihe: \(S_\infty=1\).

Avatar von 148 k 🚀

Kann man nicht einfach sagen:

Der Subtrahenden beginnt um 1 "später" als der Minuend.

Bis auf 1/1^2 hebt sich alles auf, also ist der Summenwert 1/1^2 = 1.

Bei Unendlichkeiten bin ich immer vorsichtig, da kann man sehr leicht auf die Nase fallen. Daher habe ich das lieber mathematisch sauber begründet.

Vermutlich hast du Recht und in einer Klausur würde deine Argumentation ausreichen, denke ich (bin kein Leerer).

0 Daumen

Teleskopsumme ist doch eine gute Idee!

Schreibe sie doch mal konkret für die ersten Summanden auf:

(1/1 - 1/4) + (1/4 - 1/9) + (1/9 - 1/16) + ...

Damit ist doch wohl klar, was am Ende rauskommt?


Wenn du es theoretisch sauber notieren willst, schreibe

\( \sum\limits_{k=1}^{\infty}{\frac{1}{k^2}-\frac{1}{(k+1)^2}} \)  als

\( \sum\limits_{k=1}^{\infty}{\frac{1}{k^2}}- \sum\limits_{k=1}^{\infty}{\frac{1}{(k+1)^2}} \) 

und mache hinten eine Indexverschiebung zu \(  \sum\limits_{k=2}^{\infty}{\frac{1}{k^2}} \).

Avatar von 53 k 🚀

Ein anderes Problem?

Stell deine Frage

Willkommen bei der Mathelounge! Stell deine Frage einfach und kostenlos

x
Made by a lovely community